The Student Room Group

S3

https://771a1ec81340d97ae9ed29694f73dd633b1c7c70.googledrive.com/host/0B1ZiqBksUHNYV1BTbDkxWXZCVmc/CH1.pdf

On Page 10, for part d the summation part, I have that it is 30 by doing

3 * (Var(Y)+0.25Var(X)

So I get 3(9+2) = 33 which does not match solution

Is it not this way, they use Y1+y2 etc.
Original post by L'Evil Wolf
https://771a1ec81340d97ae9ed29694f73dd633b1c7c70.googledrive.com/host/0B1ZiqBksUHNYV1BTbDkxWXZCVmc/CH1.pdf

On Page 10, for part d the summation part, I have that it is 30 by doing

3 * (Var(Y)+0.25Var(X)

So I get 3(9+2) = 33 which does not match solution

Is it not this way, they use Y1+y2 etc.


Puzzled as to why you're multiplying the Var(X) by 3.


S=Y1+Y2+Y30.5X So Var(S)=Var(Y1)+Var(Y2)+Var(Y3)+0.25Var(X)S=Y_1+Y_2+Y_3-0.5X\\\\\text{ So }Var(S)=Var(Y_1)+Var(Y_2)+Var(Y_3)+0.25Var(X)

=Var(Y)+Var(Y)+Var(Y)+0.25Var(X)=Var(Y)+Var(Y)+Var(Y)+0.25Var(X)

Hence Var(S)=3Var(Y)+0.25Var(X)Var(S)=3Var(Y) + 0.25Var(X)
(edited 7 years ago)
Reply 2
Original post by ghostwalker
Puzzled as to why you're multiplying the Var(X) by 3.


S=Y1+Y2+Y30.5X So Var(S)=Var(Y1)+Var(Y2)+Var(Y3)+0.25Var(X)S=Y_1+Y_2+Y_3-0.5X\\\\\text{ So }Var(S)=Var(Y_1)+Var(Y_2)+Var(Y_3)+0.25Var(X)

Hence Var(S)=3Var(Y)+0.25Var(X)Var(S)=3Var(Y) + 0.25Var(X)


How come it is not (Y1-0.5X)+(Y2-0.5X)+(Y3-0.5X)?
Original post by L'Evil Wolf
How come it is not (Y1-0.5X)+(Y2-0.5X)+(Y3-0.5X)?


I added an extra line to my previous post.

It's not what you've just posted because of the way S is defined. There's only one lot of X involved.
Original post by L'Evil Wolf
How come it is not (Y1-0.5X)+(Y2-0.5X)+(Y3-0.5X)?


The summation sign only applies to what's immediately following.

i=13Yi0.5X=Y1+Y2+Y30.5X\displaystyle\sum_{i=1}^{3}Y_i - 0.5X = Y_1+Y_2+Y_3-0.5X

i=13(Yi0.5X)=(Y10.5X)+(Y20.5X)+(Y30.5X)\displaystyle\sum_{i=1}^{3}(Y_i - 0.5X) = (Y_1-0.5X)+(Y_2-0.5X)+(Y_3-0.5X)
Reply 5
Original post by ghostwalker
I added an extra line to my previous post.

It's not what you've just posted because of the way S is defined. There's only one lot of X involved.


ah because X is not changing right?

thanks you for your help
Original post by L'Evil Wolf
ah because X is not changing right?

thanks you for your help


Added a second post above.

It's not because the X isn't changing.

It's really an extension of BIDMAS, or BODMAS or whatever you use. The summation is effectively a function, like log or cosine, etc.

log 6 + 7 means (log 6) + 7, not log (6+7)
Reply 7
Original post by ghostwalker
The summation sign only applies to what's immediately following.

i=13Yi0.5X=Y1+Y2+Y30.5X\displaystyle\sum_{i=1}^{3}Y_i - 0.5X = Y_1+Y_2+Y_3-0.5X

i=13(Yi0.5X)=(Y10.5X)+(Y20.5X)+(Y30.5X)\displaystyle\sum_{i=1}^{3}(Y_i - 0.5X) = (Y_1-0.5X)+(Y_2-0.5X)+(Y_3-0.5X)


Thank you very much I now understand.
Original post by L'Evil Wolf
Thank you very much I now understand.


:cool:
Reply 9
Original post by ghostwalker
The summation sign only applies to what's immediately following.

i=13Yi0.5X=Y1+Y2+Y30.5X\displaystyle\sum_{i=1}^{3}Y_i - 0.5X = Y_1+Y_2+Y_3-0.5X

i=13(Yi0.5X)=(Y10.5X)+(Y20.5X)+(Y30.5X)\displaystyle\sum_{i=1}^{3}(Y_i - 0.5X) = (Y_1-0.5X)+(Y_2-0.5X)+(Y_3-0.5X)


I know this is a while after lol but:

S = Yi - 0.5X + Y2-0.5X + Y3 - 0.5X

= 3[Yi - 0.5X]

= 3Var(Yi)+0.25Var(X)

which still yields 28 in that regard!

So it is a conincindence that both ways that you stated result in the same answer? :smile:
Reply 10
Original post by L'Evil Wolf
I know this is a while after lol but:

S = Yi - 0.5X + Y2-0.5X + Y3 - 0.5X

= 3[Yi - 0.5X]

= 3Var(Yi)+0.25Var(X)

which still yields 28 in that regard!

So it is a conincindence that both ways that you stated result in the same answer? :smile:


How did you go from the first bolded line to the second bolded line?
Original post by Zacken
How did you go from the first bolded line to the second bolded line?


Took a factor of 3 out and then applied the variance of it. I assume you consider it to be wrong?

The idea stemmed from integration, how we take out constants and then integrate if that makes sense?
Reply 12
Original post by L'Evil Wolf
Took a factor of 3 out and then applied the variance of it. I assume you consider it to be wrong?

The idea stemmed from integration, how we take out constants and then integrate if that makes sense?


By that logic, why don't you "take out the 0.5 and then applied the variance of X"?
Original post by Zacken
By that logic, why don't you "take out the 0.5 and then applied the variance of X"?


Do you mean, have:


3[Yi - 0.5X]

3/2[2Yi - X]

3/2[ 4Var(Yi) + Var(X)]

3/2 [4(9)+4]

3/2[40}
=60

which would be incorrect.
Reply 14
Original post by L'Evil Wolf
Do you mean, have:


3[Yi - 0.5X]

3/2[2Yi - X]

3/2[ 4Var(Yi) + Var(X)]

3/2 [4(9)+4]

3/2[40}
=60

which would be incorrect.


No, I'm saying, why are you doing:

Var(3(Yi - 0.5X)) = 3Var(Yi - 0.5X).
Original post by Zacken
No, I'm saying, why are you doing:

Var(3(Yi - 0.5X)) = 3Var(Yi - 0.5X).


I was doing from concepts of integration, as in the factoring out a constant, but this wouldn't apply in this different mathematical operation.
Reply 16
Original post by L'Evil Wolf
I was doing from concepts of integration, as in the factoring out a constant, but this wouldn't apply in this different mathematical operation.


I'm afraid I don't understand your point, but Ghostwalker's way is the correct version and yours is simply an (incorrect) fluke.
Original post by Zacken
I'm afraid I don't understand your point, but Ghostwalker's way is the correct version and yours is simply an (incorrect) fluke.


Okay, thank you/
Original post by L'Evil Wolf
I know this is a while after lol but:

S = Yi - 0.5X + Y2-0.5X + Y3 - 0.5X


OK, you're using the second definition of S (not the one in the original question)


= 3[Yi - 0.5X]


This is meaningless; with the Yi, rather than Y1+Y2+Y3

If you're going to follow through with the second definition, you'd have:

S=Y1+Y2+y3 - 1.5X

assuming we're dealing with just one observation of X, and not three independent ones.

Working out the variance you'd have:

Var(S) = Var(Y1)+Var(Y2)+Var(Y3)+2.25Var(X)

= 3 Var(Y) + 2.25Var(X)

which won't yield 28.

IF you had three independent observations of X instead; i.e. X1,X2,X3, you'd get

S=Y1+Y2+y3 - 0.5X1-0.5X2-0.5X3

Then Var(S) = Var(Y1)+Var(Y2)+Var(Y3)+0.25Var(X1)+0.25Var(X2)+0.25Var(X3)

=3Var(Y)+0.75Var(X)

Again, no 28.
Original post by ghostwalker
OK, you're using the second definition of S (not the one in the original question)



This is meaningless; with the Yi, rather than Y1+Y2+Y3

If you're going to follow through with the second definition, you'd have:

S=Y1+Y2+y3 - 1.5X

assuming we're dealing with just one observation of X, and not three independent ones.

Working out the variance you'd have:

Var(S) = Var(Y1)+Var(Y2)+Var(Y3)+2.25Var(X)

= 3 Var(Y) + 2.25Var(X)

which won't yield 28.

IF you had three independent observations of X instead; i.e. X1,X2,X3, you'd get

S=Y1+Y2+y3 - 0.5X1-0.5X2-0.5X3

Then Var(S) = Var(Y1)+Var(Y2)+Var(Y3)+0.25Var(X1)+0.25Var(X2)+0.25Var(X3)

=3Var(Y)+0.75Var(X)

Again, no 28.


Oh right, yes. Thank you.

Quick Reply

Latest